Đến nội dung

Khoa Linh nội dung

Có 599 mục bởi Khoa Linh (Tìm giới hạn từ 09-05-2020)



Sắp theo                Sắp xếp  

#696619 P=$(a^2-ab+b^2)(b^2-bc+c^2)(c^2-ca+a^2)\leq 1$

Đã gửi bởi Khoa Linh on 14-11-2017 - 23:07 trong Bất đẳng thức và cực trị

Cho a,b,c là các số không âm; a+b+c=3

CMR: P=$(a^2-ab+b^2)(b^2-bc+c^2)(c^2-ca+a^2)\leq 1$




#702500 P=$(a+b+c)^2$

Đã gửi bởi Khoa Linh on 28-02-2018 - 22:07 trong Đại số

bạn nói thì dễ nhưng làm thì khó lắm..... 

Nếu bạn làm được thực sự r thì mời bạn trình bày đầy đủ, sân khấu là của bạn 




#702491 P=$(a+b+c)^2$

Đã gửi bởi Khoa Linh on 28-02-2018 - 21:48 trong Đại số

Cho a,b,c>0 thỏa mãn 

$a^2+ab+b^2=25$

$b^2+bc+c^2=49$

$c^2+ca+a^2=64$

Tính giá trị P=$(a+b+c)^2$




#705813 P= $x^2+y^2+\frac{9}{x+y}$

Đã gửi bởi Khoa Linh on 13-04-2018 - 23:28 trong Bất đẳng thức và cực trị

Cho x,y T/m $x^3+y^3=2$

Tìm gtnn của biểu thức P= $x^2+y^2+\frac{9}{x+y}$

Ta có:

$2=x^3+y^3=(x+y)(x^2+y^2-xy)>0\Rightarrow x+y>0$

Ta có:

$P=x^2+y^2+\frac{9}{x+y}\geq \left (\frac{(x+y)^2}{2}+\frac{4}{x+y}+\frac{4}{x+y} \right )+\frac{1}{x+y}\geq 6+\frac{1}{x+y}$

Mặt khác: $x^2+y^2-xy\geq \frac{(x+y)^2}{4}\Rightarrow 2=(x+y)(x^2+y^2-xy)\geq \frac{(x+y)^3}{4}\Leftrightarrow x+y\leq 2$

Suy ra $P\geq \frac{13}{2}\Leftrightarrow x=y=1$




#700126 Một số BĐT sưu tầm

Đã gửi bởi Khoa Linh on 11-01-2018 - 23:36 trong Bất đẳng thức và cực trị

 

2, Cho a, b, c, d > 0 ,đặt E = $\sqrt[4]{abcd}$ Chứng minh rằng:

B=$\frac{a+d^{2}}{b}+\frac{c+a^{2}}{d}+\frac{b+c^{2}}{a}+\frac{d+b^{2}}{c}\geq 4(1+E)$

 

a/b+b/a+c/d+d/c>=4 
Cachy schwars ta có:

d^2/b+a^2/d+c^2/a+b^2/c>=a+b+c>=4E
cộng vào ta có ĐPCM




#703111 Một cano xuôi dòng từ A đến B mất 8 giờ, ngược dòng từ B về A mất 10 giờ

Đã gửi bởi Khoa Linh on 08-03-2018 - 21:25 trong Đại số

Một cano xuôi dòng từ A đến B mất 8 giờ, ngược dòng từ B về A mất 10 giờ. Hỏi một chiếc bè trôi từ A đến B mất mấy giờ? (giải bằng cách lập phương trình, nếu không được thì cách khác cũng được ạ)

Gọi vận tốc cano là a(km/h)

vận tốc dòng nước là b(km/h)

Ta có:

$\frac{AB}{a+b}=8$; $\frac{AB}{a-b}=10$ suy ra a=9b

$\frac{AB}{a-b}=10\Leftrightarrow \frac{AB}{8b}=10\Leftrightarrow \frac{AB}{b}=80$

Suy ra ra mất 80h




#700907 Một bài theo kiểu bài toán con bướm

Đã gửi bởi Khoa Linh on 29-01-2018 - 01:33 trong Hình học

Cho đường thẳng a nằm ngoài (O). Vẽ OA vuông góc với a(A thuộc a). Kẻ cát tuyến ABC và cát tuyến ADE. Gọi M, N lần lượt là giao điểm của BE và DC với a. Chứng minh rằng tam giác OMN cân

Bài toán này gọi là bài toán con bướm ngoài 
cách chứng minh vẫn thế thôi 




#700694 Min(A)=$x^2+y^2+z^2$

Đã gửi bởi Khoa Linh on 22-01-2018 - 23:08 trong Bất đẳng thức và cực trị

Cho x, y, z là các số thực thỏa mãn $x^3+y^3+z^3-3xyz=1$

Tìm Min(A)=$x^2+y^2+z^2$




#696568 min $P=\sum \frac{a}{1-a^2}$

Đã gửi bởi Khoa Linh on 13-11-2017 - 21:48 trong Bất đẳng thức và cực trị

Cho a,b,c>0 thỏa mãn $a^2+b^2+c^2=1$ Tìm min $P=\sum \frac{a}{1-a^2}$




#696572 min $P=\sum \frac{a}{1-a^2}$

Đã gửi bởi Khoa Linh on 13-11-2017 - 22:15 trong Bất đẳng thức và cực trị

Ta có

$a^{3}+\frac{2}{3\sqrt{3}}\geq a\Rightarrow a(1-a^{2})\leq \frac{2}{3\sqrt{3}}$

$\Rightarrow P=\sum \frac{a}{1-a^{2}}\geq \frac{3\sqrt{3}}{2}\sum a^{2}=\frac{3\sqrt{3}}{2}$

Cảm ơn bạn nhiều nha 




#709466 Min $\sum \frac{a^2(1-2b)}{b}$

Đã gửi bởi Khoa Linh on 28-05-2018 - 22:27 trong Bất đẳng thức và cực trị

Cho a,b,c dương và ab+bc+ca=1 

Tìm Min $\sum \frac{a^2(1-2b)}{b}$

Bài của bạn thiếu GT

Hình gửi kèm

  • Untitled.png



#699734 min $\frac{3x+8}{\sqrt{2x-1}},...

Đã gửi bởi Khoa Linh on 05-01-2018 - 13:02 trong Bất đẳng thức và cực trị

Tìm Min

$\frac{3x+8}{\sqrt{2x-1}}, \forall x > 1$

Đặt $\sqrt{2x-1}=a$

Ta có:

$\frac{3a^2+11}{2a}\geq \frac{2.\sqrt{3a^2.11}}{2a}=\sqrt{33}$




#711444 M thuộc đường thẳng cố định khi $d$ di động đi qua $M$.

Đã gửi bởi Khoa Linh on 23-06-2018 - 14:03 trong Hình học

cho $I$ nằm cố định trong đường tròn $(O)$ khác $O$ , đường thẳng d qua $I$ cắt $(O)$ tại $A$ và $B$. vẽ tiếp tuyến tại A,B cắt nhau tại $M$. CM : M thuộc đường thẳng cố định khi $d$ di động đi qua $M$.

MO cắt AB tại H, kẻ MD vuông góc với OI.

Ta có: $OI.OD=OH.OM=OA^2$ không đổi nên D cố định.

Suy ra M thuộc đường thẳng qua D và vuông góc với OI

Hình gửi kèm

  • Untitled.png



#712901 Lời giải và bình luận đề thi IMO 2018 - Tạp chí Epsilon

Đã gửi bởi Khoa Linh on 20-07-2018 - 20:46 trong Tài liệu tham khảo khác

https://drive.google...QCbqZd_1xw/view

 

 

File gửi kèm




#719832 KẾT QUẢ THI HSGQG năm 2019

Đã gửi bởi Khoa Linh on 31-01-2019 - 21:58 trong Thi HSG Quốc gia và Quốc tế

link kết quả của các tỉnh

https://drive.google...l2D5EroBAHGaOnA




#699953 Hệ thức lượng trong tam giác

Đã gửi bởi Khoa Linh on 07-01-2018 - 22:53 trong Công thức lượng giác, hàm số lượng giác

Bài 1 :

Cho $\bigtriangleup ABC thỏa mãn hệ thức  $h_{a}+h_{b}+h_{c}=9r$ .

  Chứng Minh $\bigtriangleup ABC$ đều

$h_{a}+h_{b}+h_{c}=2S.\left ( \frac{1}{a}+\frac{1}{b}+\frac{1}{c} \right )\geq \frac{2S.9}{a+b+c}=9r$

 

Dấu bằng xảy ra khi tam giác ABC đều 




#699396 Hệ phương trình của diễn đàn toán học

Đã gửi bởi Khoa Linh on 01-01-2018 - 21:44 trong Phương trình - hệ phương trình - bất phương trình

Mới kiếm thêm được một bài hay .
Giải hệ phương trình :
$ \left\{\begin{array}{l}{ x^4+y^2 =\dfrac{698}{81}} \\ {x^2+y^2+xy-3x-4y+4=0 } \end{array}\right. $

Từ PT (2) 

coi x là ẩn rồi giải delta theo y

coi y là ẩn rồi giải delta theo x

rút ta được x^4+y^2<= 698/81
Thử lại ta thấy không thỏa mãn 
=> pt vô nghiệm 




#700320 Hình học THCS

Đã gửi bởi Khoa Linh on 15-01-2018 - 00:10 trong Hình học

Cho tam giác ABC vuông tại A . Gọi I là trung điểm của AC. Qua I kẻ đường thẳng vuông góc với BC, Qua C kẻ đường thẳng vuông góc với AC , chúng cắt nhau ở E. Chứng minh: BI vuông góc với AE

Bạn tự vẽ hình nhé 

Kẻ AD vuông góc với BI; H là giao của IE và BC
áp dụng hệ thức lượng ta có:

AI^2=DI.BI; IC^2=IH.IE

=> DI.BI=IH.IE => tg IDE đồng dạng với IHB (c.g.c)

=> ED vuông góc với BI suy ra A, D, E thẳng hàng => đpcm




#699826 Hình học

Đã gửi bởi Khoa Linh on 05-01-2018 - 23:42 trong Hình học

cho (O),điểm A nằm bên ngoài (O) .Từ A vẽ 2 tiếp tuyến AB và AC.(B,C là tiếp điểm).Gọi h là giao điểm của OA và BC.

a) CM : BC vuông góc OA

b) Vẽ đường kính Bd của (O). Cm: OA // CD

c) Qua O vẽ đường thẳng vuông góc với cạnh AD tại k và cắt đường thẳng BC tại F . Cm: DF là tiếp tuyến của (O) 

Ta có tứ giác DKCF cà OKCA nội tiếp suy ra 

$\angle CDF=\angle CKF=\angle CAO=\angle OBC$

suy ra DF vuông góc với BD (q.e.d)




#713948 Hai bài hình trong đề thi trại hè Hùng Vương 2018

Đã gửi bởi Khoa Linh on 06-08-2018 - 22:55 trong Tài liệu tham khảo khác

Tham  khảo tại đây: https://khoalinhmath...ai-he-hung.html

 

Download: https://drive.google...F9fr6SeVCf/view

 

Hai bài hình học phẳng thi trại hè Hùng Vương 2018-01.jpg




#700245 Giải pt: 2$\sqrt{2x+4}+ 4\sqrt{2-x}=\...

Đã gửi bởi Khoa Linh on 13-01-2018 - 18:25 trong Phương trình, hệ phương trình và bất phương trình

Giải pt: 2$\sqrt{2x+4}+ 4\sqrt{2-x}=\sqrt{9x^2+16}$

26239407_139564680056973_171586139739792

Nguồn: Bùi Quyết




#700319 Giải pt: 2$\sqrt{2x+4}+ 4\sqrt{2-x}=\...

Đã gửi bởi Khoa Linh on 15-01-2018 - 00:01 trong Phương trình, hệ phương trình và bất phương trình

PT vô nghiệm?

Bạn tự giải tiếp ra đi 

nghiệm là căn(32/9)




#698470 GIÚP ĐỠ: $\frac{1}{a^2+b}+\frac{1...

Đã gửi bởi Khoa Linh on 17-12-2017 - 17:57 trong Bất đẳng thức và cực trị

Bước $\sqrt{c.c.b}+\sqrt{a.c.c}+\sqrt{b.b.a} \leq \frac{c+c+b+a+c+c+b+b+a}{3}$ không đúng rồi bạn ạ.

Phải là căn bậc 3 mới áp dụng được BĐT AM-GM ở đây.

 

BĐT này sai rồi. Với $a=b=c=2$, $\frac{1}{a^2+b}+\frac{1}{b^2+c}+\frac{1}{c^2+a}=\frac{1}{2}$, $\frac{a+b+c}{2abc}=\frac{3}{8}$

sorry mình nhầm




#698439 GIÚP ĐỠ: $\frac{1}{a^2+b}+\frac{1...

Đã gửi bởi Khoa Linh on 17-12-2017 - 10:02 trong Bất đẳng thức và cực trị

NHỜ CÁC BẠN GIẢI GIÚP MÌNH 3 BÀI NÀY VỚI:

1) cho a,b,c là độ dài ba cạnh của tam giác. chứng minh rằng: $\frac{1}{a^2+b}+\frac{1}{b^2+c}+\frac{1}{c^2+a}\leq \frac{a+b+c}{2abc}$

2) Cho a, b, c là ba số thực không âm và $a+b+c=1$. CMR: $\sqrt{7a+9}+\sqrt{7b+9}+\sqrt{7c+9}\geq 10$

3) Cho a, b, c dương. CMR: $\frac{a^3}{b}+\frac{b^3}{c}+\frac{c^3}{a}\geq a\sqrt{ac}+b\sqrt{ba}+c\sqrt{cb}$

XIN CẢM ƠN CÁC BẠN

Câu 1:

25287044_517246245316045_1764584949_n.pn




#698438 GIÚP ĐỠ: $\frac{1}{a^2+b}+\frac{1...

Đã gửi bởi Khoa Linh on 17-12-2017 - 09:57 trong Bất đẳng thức và cực trị

NHỜ CÁC BẠN GIẢI GIÚP MÌNH 3 BÀI NÀY VỚI:

1) cho a,b,c là độ dài ba cạnh của tam giác. chứng minh rằng: $\frac{1}{a^2+b}+\frac{1}{b^2+c}+\frac{1}{c^2+a}\leq \frac{a+b+c}{2abc}$

2) Cho a, b, c là ba số thực không âm và $a+b+c=1$. CMR: $\sqrt{7a+9}+\sqrt{7b+9}+\sqrt{7c+9}\geq 10$

3) Cho a, b, c dương. CMR: $\frac{a^3}{b}+\frac{b^3}{c}+\frac{c^3}{a}\geq a\sqrt{ac}+b\sqrt{ba}+c\sqrt{cb}$

XIN CẢM ƠN CÁC BẠN

Câu 3:

25520836_517245261982810_1763229441_o.pn